Jump to content
  • Member Statistics

    17,502
    Total Members
    7,904
    Most Online
    Weathernoob335
    Newest Member
    Weathernoob335
    Joined

Greenhouse Effect 101


blizzard1024

Recommended Posts

Decided to start a new thread to discuss some basic greenhouse effect science. This is because there has been a colorful discussion of this on the 2016 Global Temperature thread. 

The problem that has been debated is how greenhouse gases affect our entire Earth System without violating the conservation of energy.  

I will add another way of describing this... 

If you increase greenhouse gases in the troposphere, you reduce the outgoing long wave radiation (OLR) in the troposphere. This means that there is less radiation reaching the higher levels of the atmosphere causing cooling. This leads to a warmer troposphere and cooler stratosphere and layers above. In this way, energy is conserved...  

Link to comment
Share on other sites

Your accounting of the radiation coming from various sources has never been complete.

 

If you increase GHG concentrations, the percentage of outgoing IR from the earth drops.  But that is not the only thing that happens.  The earth's surface temperature will increase because of this, as will the temperatures of the layers that are now absorbing more IR so there is no drop.  There can be no drop, as the earth must emit the same amount of energy due to conservation of energy.  

Link to comment
Share on other sites

If the sun provides a constant source of energy to the Earth system (land, oceans, and entire atmosphere), greenhouse gases absorb IR that would otherwise go to space and lead to a warm lower atmosphere. If the atmosphere in its entirety gains any heat from this, you have added energy. How can it be otherwise? If this happens you violate the conservation of energy. The Sun-Earth System is a closed system. Where else do we get 99.99999% of our energy from?  One can argue may be a little comes from geothermal and a little from star light but these are soooooo small. How is the Earth system - Sun not a close system? That is where I might be misunderstanding this problem...

When I learned radiative transfer at the graduate level, this was pretty well known that GHGs don't add energy to the Earth system. They warm the troposphere and cool above. Its hard to believe that basic laws of science have changed in this new era of science???  

1 hour ago, drstuess said:

Read the first law of thermodynamics and do the basic calculations which have been outlined before for yourself. There may be some cooling, but conservation of energy in no way requires it to be equal in magnitude.

 How can this be? you are allowing GHGs to create energy??? They don't create energy they just intercept photons and redirect them....  

Link to comment
Share on other sites

5 hours ago, blizzard1024 said:

If the sun provides a constant source of energy to the Earth system (land, oceans, and entire atmosphere), greenhouse gases absorb IR that would otherwise go to space and lead to a warm lower atmosphere. If the atmosphere in its entirety gains any heat from this, you have added energy. How can it be otherwise? If this happens you violate the conservation of energy. The Sun-Earth System is a closed system. Where else do we get 99.99999% of our energy from?  One can argue may be a little comes from geothermal and a little from star light but these are soooooo small. How is the Earth system - Sun not a close system? That is where I might be misunderstanding this problem...

When I learned radiative transfer at the graduate level, this was pretty well known that GHGs don't add energy to the Earth system. They warm the troposphere and cool above. Its hard to believe that basic laws of science have changed in this new era of science???  

 How can this be? you are allowing GHGs to create energy??? They don't create energy they just intercept photons and redirect them....  

1. The Earth and sun is not a closed system. The earth radiates heat out to the great beyond 

2. While the initial Q in from the sun is constant. The outgoing Q from the earth is not constant and depends on 1. GHG'S and 2. The earth's temperature. 

Adding greenhouse gases reduce Q out for the period of time until temperature dependant radiation increases. This results in a non zero incoming q to the sysyem. Applying the first law means that W and/or deltaE (temperature) must increase to conserve energy.

This is applying the first law of thermodynamics ("conservation of energy") to the system. Yes the upper atmosphere may cool, however conservation of energy at this scale, with the simplified model outlined, does not necessitte it match warming. I will post the basic first law application my feeble non-graduate radiative heat transfer mind sketched earlier. The first frame is before addiotnal ghg, the second is with additional ghg ( but transient state), and third is equilibrium with ghg. BTW the first law hasn't changed.

 

Edit: I think you are stuck in your application of the first law and what the system is. Phrasing it how you are, yes if the Earth is warming up, something else must cool appropriately. That something else is the rest of the solar system, universe, etc and not just the upper atmosphere. 

Resized_20170114_110123001.jpeg

Link to comment
Share on other sites

Its funny because when I learned radiative transfer at the graduate level the first thing we did was create a series of simple atmospheric models to understand the GHE.  It was very clear that there was no violation of conservation of energy because while the flux in and the flux out is the same when equilibrium is reached, the reservoir of energy is what changes.  The energy that now resides in the resevoir is simply not out in space.  The earth taking longer to radiate energy out to space isn't a violate of converservation of energy.

 

 

Link to comment
Share on other sites

Ok. So you are both saying that greenhouse gases increase the energy of the entire atmosphere even if there is some cooling above the troposphere. 

The transient state as you both speak of is when there is an imbalance in the entire Earth system. The imbalance that would cause warming of the entire atmosphere and Earth system would be less radiation leaving than is coming in leaving a positive energy imbalance. So, by adding greenhouse gases you create energy. This can't be. The imbalance only can be in the lower atmosphere to conserve energy. How can you say there is more cooling to balance the warming by the Earth radiating to space? If the Earth has more greenhouse gases and the planet's atmosphere warms it radiates more strongly which leads to cooling. The cooling seen would be the upper levels of the atmosphere. Greenhouse gases cannot create energy. They simply redirect photons in all directions. Some reach the Earth and warm while some radiate to outer space more strongly. Net effect is zero gain in energy of the entire Earth system. Again greenhouse gases redirect photons of IR. That's all they do. You can't warm an entire atmosphere by this process. 

drstuess- you equation in the transient state has a delta E > 0 which is creating energy. 

Msalgado- 

4 hours ago, Msalgado said:

The earth taking longer to radiate energy out to space isn't a violate of converservation of energy.

This is the case and because of this, it cools at high levels. the energy reservoir you talk about is the Oceans, Land and Troposphere which I agree would warm up. But there has to be compensating cooling at high levels.

This is a tough concept and even PHD scientists struggle with it. Appreciate the civil discussion. I am going to contact a PHD scientist I know in radiative transfer and thermodynamics and see if I can get a better explanation of this or some clarification.  

thanks. 

 

 

Link to comment
Share on other sites

Yes delta E is non zero in the system, as there is an imbalance in the flux of q and w in the system during the transient state. That does not mean that energy is created; Q and W are transferors of energy and their values account for the increase in E.  It is the first law of thermodynamics!

Additionally, the Earth doesn't cool off because it radiates more. The temperature simply increases until the temperarure dependant radiation and other outgoing Q fluxes balance against the incoming q flux.  At that point the qdot term is zero in the first law and , assuming W is zero, deltaE is zero.

Link to comment
Share on other sites

6 hours ago, drstuess said:

Yes delta E is non zero in the system, as there is an imbalance in the flux of q and w in the system during the transient state. That does not mean that energy is created; Q and W are transferors of energy and their values account for the increase in E.  It is the first law of thermodynamics!

Additionally, the Earth doesn't cool off because it radiates more. The temperature simply increases until the temperarure dependant radiation and other outgoing Q fluxes balance against the incoming q flux. At that point the qdot term is zero in the first law and , assuming W is zero, deltaE is zero.

Delta E is change in Energy? Power? or power/area?  I assume E for energy. Q and W redistribute energy. I get that so they don't change the total energy. But, in order for energy to be conserved if some system gains energy another has to lose energy. So your "transport" or flux terms simply move energy. So where is the energy that is lost?  This energy is moving from one place and "collecting" in another. The Earth system I argue is a closed system with the sun. Others disagree which is fine.  If the Earth system - Sun is a closed system than delta E can't be non-zero for the entire Earth system including the whole atmosphere.  

"Additionally, the Earth doesn't cool off because it radiates more. The temperature simply increases until the temperarure dependant radiation and other outgoing Q fluxes balance against the incoming q flux."  If any object radiates or emits more all else remaining equal, it loses energy and hence cools. I think what you are trying to say is that when you increase GHGs, the effective radiating layer rises to cooler levels of the troposphere. Hence the emission is reduced because cool air has lower emission. If the emission is reduced, for the same incoming SW radiation, the temperature must rise to increase emissions to balance what the incoming solar or SW radiation is. This does indeed warm the tropopshere. However, in the stratosphere, temperatures increase with height and hence GHGs lead to cooling. Emission increases with height!! 

GHGs do not create energy. I know we can agree on that. They slow the transfer of energy through the Earth system leading to warming down low(troposphere) and cooling way up (I mean stratosphere to exosphere) way way up!  My struggle is how is the Earth system - Sun not a closed system? The earth receives practically all its energy from the sun. So how can this not be a closed system? If someones has a good explanation for this, please share....   Thanks. 

   

Link to comment
Share on other sites

1 hour ago, blizzard1024 said:

 My struggle is how is the Earth system - Sun not a closed system? The earth receives practically all its energy from the sun. So how can this not be a closed system? If someones has a good explanation for this, please share....   Thanks. 

   

Others have explained well but I will try one more time. You need to include space to close the system because the earth radiates to space. Energy comes from the sun through earth and then to space. The increase in the earth's energy from ghg is balanced entirely by less energy radiated to space.  The tank analogy developed on the other thread is a good one. In the tank case the increase in water in the tank is balanced by less water going out the drain. If you only include the faucet and the tank the system is not closed.

Link to comment
Share on other sites

The Earth radiates energy into space in all directions, how can it be closed?

What loses energy is the "outside system", which depends on what you define as the system. Given the 101 nature of the thread it is a single layer model, so the outside is space.

Going to multilayer model with individual emmissivity s etc gets significantly more complicated in transient states. Yes, starting at the bottom any positive q flux is countered by negative q flux immediately above, however the exact amount depends on its absorbtivity. the exact relationships, especially in transient are pretty complicated as you must account for varying density as well, which will complicate the temperature change.

 

Also to the other comment on settled science education haha. I am just some Joe schmoo taping on my phone; I don't think this represents where the actual science is at.

 

Link to comment
Share on other sites

blizzard1024, if you would, could you let me know what you think about this experiment? It's a simpler one than the one I originally posted in the 2016 temperatures thread, and I do not know if you've seen this one yet.

Experiment 2a: Start with an empty tank, with a small hole in the bottom. Let water flow into the top of the tank via a tap at a (near-) constant rate, such that the rate at which the water flows into the top is a little faster at first than the rate at which it exits the bottom of the tank. I think we can all agree that the tank would begin to fill with water, as the inflow rate is greater than the outflow rate. But what will happen after that? Will the tank continue to fill at the same rate perpetually? Or will something else happen? To me, it is clear that, as the water level increases, the water pressure in the bottom of the tank increases, increasing the outflow rate. Eventually, the outflow rate will match the inflow rate. At that point, the water level has reached an "equilibrium" level. Can we agree that this is what would happen in this experiment?

Link to comment
Share on other sites

49 minutes ago, Mallow said:

blizzard1024, if you would, could you let me know what you think about this experiment? It's a simpler one than the one I originally posted in the 2016 temperatures thread, and I do not know if you've seen this one yet.

Start with an empty tank, with a small hole in the bottom. Let water flow into the top of the tank via a tap at a (near-) constant rate, such that the rate at which the water flows into the top is a little faster at first than the rate at which it exits the bottom of the tank. I think we can all agree that the tank would begin to fill with water, as the inflow rate is greater than the outflow rate. But what will happen after that? Will the tank continue to fill at the same rate perpetually? Or will something else happen? To me, it is clear that, as the water level increases, the water pressure in the bottom of the tank increases, increasing the outflow rate. Eventually, the outflow rate will match the inflow rate. At that point, the water level has reached an "equilibrium" level. Can we agree that this is what would happen in this experiment?

i note your other post about ME.......i never disagreed with what you just wrote above.......i accepted that and dont really know why you keep asking me to accept that i DID long ago........BUT that is NOT your experiment, AFTER we reach that point you close the bottom opening by a tiny bit and claim that causes a build up in the tank which it would NOT, because as i explained in the real world closing the aperture a tiny bit SPEEDS UP the water leaving and the other poster even made that point.......the existing pressure in the tank is sufficient to speed up the flow when a tiny change happens at the bottom......just like pinching a water hose does NOT cause any backup of water, the water IMMEDIATELY squirts further and faster and the SAME amount of water leaves the hose......just as the same amount of water would leave your tank........YOU are trying to devise an experiment to show there is some LAG in thermodynamics that the temperature has to rise a good bit before the outflow increases in speed and that is FALSE.....the moment the differential between the warm body and cold body increases the movement of energy speeds up, there is NO lag time waiting to reach some triggering point at a higher temp..........PLEASE stop attempting to insult me and discuss the science........ i also addressed the concept of "equilibrium" in that thread, that point is a MOVING point that cant ever be found because of the countless factors involved that most all are constantly changing, there IS a hypothetical point of balance but NOT in the real world, the entire record shows up and down cycles in the temperatures which is PROOF there is NO BALANCE, the implication i see is your side trying to claim the earth was in this pristine state of equilibrium and then mean ole humans upset the balance.......but the FACT is there is NO balance and no such state in the real world.

Link to comment
Share on other sites

1 hour ago, BillT said:

i note you other post about ME.......i never disagreed with what you just wrote above.......i accepted that and dont really know why you keep asking me to accept that i DID long ago........BUT that is NOT your experiment, AFTER we reach that point you close the bottom opening by a tiny bit and claim that causes a build up in the tank which it would NOT, because as i explained in the real world closing the aperture a tiny bit SPEEDS UP the water leaving and the other poster even made that point.......the existing pressure in the tank is sufficient to speed up the flow when a tiny change happens at the bottom......just like pinching a water hose does NOT cause any backup of water, the water IMMEDIATELY squirts further and faster and the SAME amount of water leaves the hose......just as the same amount of water would leave your tank........YOU are trying to devise an experiment to show there is some LAG in thermodynamics that the temperature has to rise a good bit before the outflow increases in speed and that is FALSE.....the moment the differential between the warm body and cold body increases the movement of energy speeds up, there is NO lag time waiting to reach some triggering point at a higher temp..........PLEASE stop attempting to insult me and discuss the science........ 

Although my question was addressed to blizzard1024, thank you for responding. It was not clear to me that you had agreed with this particular experimental design, as your responses always seemed to be about something different.

Please do not assume where I am going with this experiment. I was not going to suggest that the next step was to change the aperture size after reaching an equilibrium, as you state. As you can see, it makes the discussion much more difficult when assumptions about intentions are made.

I still absolutely disagree with the notion that decreasing the size of the aperture does not change the water flow through the aperture, as it is completely unphysical and unintuitive, and you have not demonstrated any actual evidence that this occurs (you have claimed it's true in the hose example, and I strongly disagree with the assertion that flow rate is unchanged even in that example--regardless, that experiment does not have any relevance towards mine). However, let us ignore that point of contention for the moment, since it is not relevant to the experiment we are currently discussing.

Since we agree that the experiment I have described above would lead to the outcome I have described above (that is, starting from an empty tank, the water would fill up until it reaches an equilibrium level where the increased water pressure in the bottom allows the flow rate to match the constant incoming flow rate), my next question is as follows.

Experiment 2b: Start with an empty tank, with a very slightly smaller hole in the bottom than in experiment 2a. Let water flow into the top of the tank via a tap at the same (near-) constant rate as in experiment 2a. In experiment 2b, because the hole in the bottom of the tank is slightly smaller than in experiment 2a, the initial outflow rate is slightly lower. Therefore, the rate at which the water flows into the top is still a little faster at first than the rate at which it exits the bottom of the tank, and the initial differential is slightly larger in experiment 2b than in 2a. I think we can all agree that the tank would again begin to fill with water, as the inflow rate is still greater than the outflow rate. But what will happen after that? To me, it is clear that, as the water level increases, the water pressure in the bottom of the tank increases, increasing the outflow rate, just as in experiment 2a. Eventually, the outflow rate will match the inflow rate, however, since the hole in the bottom is smaller, a higher pressure will be needed to reach this equilibrium level. At that point, the water level has reached an "equilibrium" level which is slightly higher than in experiment 2a. Does this seem reasonable?

Link to comment
Share on other sites

1 minute ago, Mallow said:

Although my question was addressed to blizzard1024, thank you for responding. It was not clear to me that you had agreed with this particular experimental design, as your responses always seemed to be about something different.

Please do not assume where I am going with this experiment. I was not going to suggest that the next step was to change the aperture size after reaching an equilibrium, as you state. As you can see, it makes the discussion much more difficult when assumptions about intentions are made.

Since we agree that the experiment I have described above would lead to the outcome I have described above (that is, starting from an empty tank, the water would fill up until it reaches an equilibrium level where the increased water pressure in the bottom allows the flow rate to match the constant incoming flow rate), my next question is as follows.

Start with an empty tank, with a very slightly smaller hole in the bottom than in the previous experiment. Let water flow into the top of the tank via a tap at the same (near-) constant rate as in the previous experiment. In this second experiment, because the hole in the bottom of the tank is slightly smaller than the previous experiment, the initial outflow rate is slightly lower. Therefore, the rate at which the water flows into the top is still a little faster at first than the rate at which it exits the bottom of the tank, and the initial differential is slightly larger. I think we can all agree that the tank would again begin to fill with water, as the inflow rate is still greater than the outflow rate. But what will happen after that? To me, it is clear that, as the water level increases, the water pressure in the bottom of the tank increases, increasing the outflow rate, just as before. Eventually, the outflow rate will match the inflow rate, however, since the hole in the bottom is smaller, a higher pressure will be needed to reach this equilibrium level. At that point, the water level has reached an "equilibrium" level which is slightly higher than in the first experiment. Does this seem reasonable?

that experiment does not apply to the atmosphere in any way..........and that was NOT the original premise, clearly the original had a lessening of the bottom hole, and i corrected your claim as to what would happen......so NOW you remove that error rendering your entire experiment meaningless in use to show how the atmosphere functions.........my POINT on your original experiment with the slight closing of the hole(that you now dont do) was the existing pressure would speed up the flow through the hole because the tiny closing increases the pressure at THAT single location and that increased pressure is released by making the water go faster and further IMMEDIATELY...........i will post then and then go find your original "experiment".

Link to comment
Share on other sites

3 minutes ago, BillT said:

that experiment does not apply to the atmosphere in any way..........and that was NOT the original premise, clearly the original had a lessening of the bottom hole, and i corrected your claim as to what would happen......so NOW you remove that error rendering your entire experiment meaningless in use to show how the atmosphere functions.........my POINT on your original experiment with the slight closing of the hole(that you now dont do) was the existing pressure would speed up the flow through the hole because the tiny closing increases the pressure at THAT single location and that increased pressure is released by making the water go faster and further IMMEDIATELY...........i will post then and then go find your original "experiment".

Again, please do not make assumptions about my intentions. We are not discussing my original experiment at the moment.

Do you agree with my conclusions about my second simpler experiment above?

Link to comment
Share on other sites

1 minute ago, Mallow said:

Again, please do not make assumptions about my intentions. We are not discussing my original experiment at the moment.

Do you agree with my conclusions about my second experiment?

i already DID and this is the problem you demand i do things i already DID......why ASK me now if i accept the above i clearly said YES I DO above.......and here is your original experiment

"

You have a tank of water. In the bottom of the tank is a hole. The rate at which water drains out of this hole depends on the size of the hole, and on the water pressure, which is proportional to the depth of water in the tank. Additionally, you have a faucet adding water at a near-constant rate. Now, if the water level in the tank remains the same, it means that the faucet is adding water at the same rate at which the hole is draining it. I believe this is all relatively straightforward so far.

Now, at time t=0, we decrease the size of the hole in the bottom of the tank. Since the water level is initially unchanged, but the hole is smaller, we have less water flowing out the bottom than is being added at the top. This causes the water level in the tank to increase. Eventually, the water level in the tank will be high enough that the increased pressure at the bottom of the tank will counteract the effect of the decreased hole. This is the "new equilibrium" water level in the tank, the new level at which the incoming water balances the water draining out the bottom, and we will call the time at which this new equilibrium is reached t = te.

so why are you NOW altering your position?   and i note another thing, YOU clearly say a "faucet" is the source of your inflow yet you later claimed after i pointed out your error that MY use of a "faucet" with the water hose complicates things too much for you.......so now by altering your claim are you making an admission that i was correct in showing your original error and you now remove that error?   and the POINT of your experiment was to show the effects of pressure changes so whether the pressure comes from a column of water or the city water lines the pressure and how it reacts are the SAME.....so please dont claim i am non responsive or havent discussed these things at length..........so now the other poster on your side claiming i am not posting in good faith is shown to be WRONG, it is YOU that ids altering your claims in some bizarre attempt to force me into agreeing with components that i already accepted as givens when this first began.

Link to comment
Share on other sites

15 minutes ago, BillT said:

i already DID and this is the problem you demand i do things i already DID......why ASK me now if i accept the above i clearly said YES I DO above.......and here is your original experiment

I'm sorry, I was not clear. I was not talking about the first simpler experiment, I mean my second simpler experiment, in which I claim that the final equilibrium level would be higher than in the first simpler experiment. To make things cleaner, let's call my original experiment "Experiment 1", and then let's call the two simpler experiments "Experiments 2a and 2b". We clearly disagree about experiment 1. However, as you noted, you have agreed with my conclusions about experiment 2a. Do you also agree with my conclusions about experiment 2b?

Link to comment
Share on other sites

29 minutes ago, Mallow said:

I'm sorry, I was not clear. I was not talking about the first simpler experiment, I mean my second simpler experiment, in which I claim that the final equilibrium level would be higher than in the first simpler experiment. To make things cleaner, let's call my original experiment "Experiment 1", and then let's call the two simpler experiments "Experiments 2a and 2b". We clearly disagree about experiment 1. However, as you noted, you have agreed with my conclusions about experiment 2a. Do you also agree with my conclusions about experiment 2b?

NO and please end this here, there is nothing to be gained and this has strayed far away from the topic......i also state clearly i do not appreciate the implication that you had to make things more simple for me to understand......i understood EVERYTHING you wrote nothing needed to be simplified.

Link to comment
Share on other sites

1 minute ago, BillT said:

NO and please end this here, there is nothing to be gained and this has strayed far away from the topic......i also state clearly i do not appreciate the implication that you had to make things more simple for me to understand......i understood EVERYTHING you wrote nothing needed to be simplified.

For the third time, please do not assume my intentions.

What is it that you specifically disagree with in my conclusions for experiment 2b?

Link to comment
Share on other sites

9 minutes ago, JC-CT said:

Oh lord. So the same amount of water would exit a cylinder with no bottom whatsoever as it would a cylinder with a pin-sized hole at the bottom, because the water pressure would equal out?

Hahahahahahahahahahahaha

deleted, because responding would continue something i am finished discussing

Link to comment
Share on other sites

22 hours ago, drstuess said:

The Earth radiates energy into space in all directions, how can it be closed?

What loses energy is the "outside system", which depends on what you define as the system. Given the 101 nature of the thread it is a single layer model, so the outside is space.

Going to multilayer model with individual emmissivity s etc gets significantly more complicated in transient states. Yes, starting at the bottom any positive q flux is countered by negative q flux immediately above, however the exact amount depends on its absorbtivity. the exact relationships, especially in transient are pretty complicated as you must account for varying density as well, which will complicate the temperature change.

 

Also to the other comment on settled science education haha. I am just some Joe schmoo taping on my phone; I don't think this represents where the actual science is at.

Thanks good Dr. for the expression of humility.  Quite refreshing.  I only meant to point out that nothing is as simple as it seems and that there is always a reason to debate an issue if for nothing more than to make folks reexamine old assumptions and perhaps look at issues in new and refreshing ways.  Or maybe the debate merely serves to provide new confidence in old assumptions.

Heresy, I know.

 

22 hours ago, drstuess said:

 

 

Link to comment
Share on other sites

 

Since  there is continuing disagreement regarding the physics associated with a bucket of water with a hole in it, let's have some Saturday morning fun in the Greenhouse Effect 101 thread with a simpler question.   

Which of the following pictures categorically depicts the emission of the dangerous "pollutant" CO2 into the atmosphere? 

 

58F7636D-77D7-4716-A2CC74D9AA5EF5D4.jpg?
Credit: iStockphoto

iStock_000014437621XSmall.jpg?itok=Deym-

  ADVERTISEMENT | 
REPORT AD
 

 

 

 

Link to comment
Share on other sites

17 minutes ago, Jack Frost said:

 

Since  there is continuing disagreement regarding the physics associated with a bucket of water with a hole in it, let's have some Saturday morning fun in the Greenhouse Effect 101 thread with a simpler question.   

Which of the following pictures categorically depicts the emission of the dangerous "pollutant" CO2 into the atmosphere? 

 

58F7636D-77D7-4716-A2CC74D9AA5EF5D4.jpg?
Credit: iStockphoto

iStock_000014437621XSmall.jpg?itok=Deym-

  ADVERTISEMENT | 
REPORT AD
 
 

 

 

 

None. CO2 is an invisible gas. None of the photos "categorically" depicts the emission of CO2. On the latter, we don't know if the athlete is inhaling or exhaling. 

Link to comment
Share on other sites

1 hour ago, donsutherland1 said:

None. CO2 is an invisible gas. None of the photos "categorically" depicts the emission of CO2. On the latter, we don't know if the athlete is inhaling or exhaling. 

Good point Don.  I agree that nothing is ever as obvious as it may seem.

Like the IPCC confidence levels and "lines of evidence".... 

By the way, I did have to smile when I read your post.

Link to comment
Share on other sites

Getting back to the basic greenhouse effect, if the atmosphere gains greenhouse gases, they will absorb more outgoing long wave radiation and warm the Earth.

However, this will reduce the amount of OLR going to higher levels and hence there will be cooling above. This simple statement shows that a planet can't gain energy from its own

atmospheric gases.  The amount of energy that comes in, is balanced by the energy that leaves the system. How the energy is distributed is what GHGs do. There is no way the earth system including the ENTIRE atmosphere can gain energy. It can't happen. If you look at Venus, the higher atmosphere is much colder than the earth's because of the runaway greenhouse effect. See below...

 

Robinson_Catling_2013_f1.png

 

Link to comment
Share on other sites

Archived

This topic is now archived and is closed to further replies.

  • Recently Browsing   0 members

    • No registered users viewing this page.
×
×
  • Create New...